The Student Room Group

Algebra tricky question

image.jpeg
Very confused on parts 1 and 2 of this, not sure of the proof at all,
And how do I go abouts part 3?
Thankyou for any help in advance

Scroll to see replies

Reply 1
Original post by maths10101
image.jpeg
Very confused on parts 1 and 2 of this, not sure of the proof at all,
And how do I go abouts part 3?
Thankyou for any help in advance


Can you post a proper picture - this is cut off a little. What are your thoughts on it?
Reply 2
Original post by Zacken
Can you post a proper picture - this is cut off a little. What are your thoughts on it?


image.jpeg
Sorry about that and ignore part c, I've figured fhat one out..
For part a, I'm completely clueless of how to carry it out
For part b, would you find the orders of all elements of A^6? And use this to show that g is 1? There must be a quicker way of doing this as this method is quite time consuming
Reply 3
Original post by maths10101
image.jpeg
Sorry about that and ignore part c, I've figured fhat one out..
For part a, I'm completely clueless of how to carry it out
For part b, would you find the orders of all elements of A^6? And use this to show that g is 1? There must be a quicker way of doing this as this method is quite time consuming


For a) and b) surely you could just apply Lagrange?
As for b) I wouldn't want to find all the elements of A6, there are 360 of them.
(edited 8 years ago)
Reply 4
Original post by joostan
For a) and b) surely you could just apply Lagrange?
As for b) I wouldn't want to find all the elements of A6, there are 360 of them.


So for a) i've seen that since the cardinality of the subgroup divides the cardinality of the group (S-12), it must be a subgroup by lagrance theorem,,,,does this seem okay?
and for part b....i'm not sure how to prove that
Reply 5
Original post by maths10101
So for a) i've seen that since the cardinality of the subgroup divides the cardinality of the group (S-12), it must be a subgroup by lagrance theorem,,,,does this seem okay?
and for part b....i'm not sure how to prove that


I don't follow what you're saying for a), Lagrange's Theorem states that the order of a subgroup must divide the order of the group.
What is the order of S12S_{12}? What can you then say about the order of the subgroups of S12S_{12}?

As for (b) how many elements of A6A_6 are there?
Now if you suppose that an element gg of order 7 exists, what can you say about the set 1,g,...,g61,g,. . .,g^6?
Reply 6
Original post by joostan
I don't follow what you're saying for a), Lagrange's Theorem states that the order of a subgroup must divide the order of the group.
What is the order of S12S_{12}? What can you then say about the order of the subgroups of S12S_{12}?

As for (b) how many elements of A6A_6 are there?
Now if you suppose that an element gg of order 7 exists, what can you say about the set 1,g,...,g61,g,. . .,g^6?


The order of S_12 is 479001600
and i'm not finding anything about the order of the subgroups online?
Reply 7
Original post by maths10101
The order of S_12 is 479001600
and i'm not finding anything about the order of the subgroups online?


It has order 12! - does 17 divide that? Prove that it doesn't.
Reply 8
Original post by Zacken
It has order 12! - does 17 divide that? Prove that it doesn't.

Ahhh brilliant! Any ideas on part 2 mate?
Reply 9
Original post by maths10101
The order of S_12 is 479001600
and i'm not finding anything about the order of the subgroups online?


You know that the order of S12S_{12} is 12!12!.
You want to show that there is no subgroup of order 1717.
If a group of order nn has a subgroup of order kk, then we must have knk|n by Lagrange.
Apply this to this case.

Again, for the second question, the order of an element must divide the order of the group by Lagrange.
Thus if g7=1g^7=1 then what are the possible orders of gg?
(edited 8 years ago)
For Part 3) use the method of successive squaring.
Reply 11
Original post by fishyfishy1999
For Part 3) use the method of successive squaring.


He's already done part 3. :-)
Original post by Zacken
He's already done part 3. :-)


Oops. The answer is 10 for what it is worth.
Reply 13
Original post by fishyfishy1999
Oops. The answer is 10 for what it is worth.


Given that he's already done it, mind posting an outline of how you do it? I'm interested in knowing and I haven't heard of your method before. If it's not too much trouble, that is.
Original post by Zacken
Given that he's already done it, mind posting an outline of how you do it? I'm interested in knowing and I haven't heard of your method before. If it's not too much trouble, that is.



It's a pain to write down in this box. Just Google Successive Squaring Algorithm for computing a^k mod m.

Quite straightforward.

End up with 18 14 18 14 18 14 18 18 7 10 7 7 10 14 7 10 14 18 10 as the non-zero remainders that are multiplied together to give 439205094557429760000 mod 31 which is 10.
Reply 15
Original post by fishyfishy1999
It's a pain to write down in this box. Just Google Successive Squaring Algorithm for computing a^k mod m.

Quite straightforward.

End up with 18 14 18 14 18 14 18 18 7 10 7 7 10 14 7 10 14 18 10 as the non-zero remainders that are multiplied together to give 439205094557429760000 mod 31 which is 10.


Will do so demain in the morning, thanks.
Reply 16
ImageUploadedByStudent Room1456746198.061610.jpg

Hey, was just wondering if I've done this question correctly for part b?


Posted from TSR Mobile

@zacken
Reply 17
Also, surely Fermat's Little theorem would be easier to apply to work out part c?
@Zacken @fishyfishy1999
Original post by maths10101
Also, surely Fermat's Little theorem would be easier to apply to work out part c?
@Zacken @fishyfishy1999


How does Fermat's Little Theorem (FLT) help you compute this?

FLT is often used in tandem with successive squaring to checking if a number is composite without finding the factors themselves.
Reply 19
@fishyfishy1999 it works perfectly here..get the same answer ImageUploadedByStudent Room1456749900.744237.jpg


Posted from TSR Mobile

Quick Reply

Latest